Can Derivative Work?

Algebra Level 5

{ x + y + z = 4 x 2 + y 2 + z 2 = 6 \large{\begin{cases} x+y+z=4 \\ x^2+y^2+z^2 =6 \end{cases}}

Real numbers x , y x,y and z z satisfy the system of equations above.

Let P = x 3 + y 3 + z 3 P =x^3+y^3+z^3 . If the difference between the maximum value of P P and the minimum value of P P can be expressed as a b \dfrac ab , where a a and b b are coprime positive integers, find a + b a+b .

Bonus: Generalize the result for x + y + z = m x+y+z=m and x 2 + y 2 + z 2 = n x^2+y^2+z^2=n .


Source: Grade 10 Gifted Math Students Selection Test, University of Education, HCMC, Vietnam.


The answer is 13.

This section requires Javascript.
You are seeing this because something didn't load right. We suggest you, (a) try refreshing the page, (b) enabling javascript if it is disabled on your browser and, finally, (c) loading the non-javascript version of this page . We're sorry about the hassle.

5 solutions

Tran Quoc Dat
Jun 9, 2016

My solution bases on Jerry Han Jia Tao 's one, but simpler.

From the system we have x 2 + y 2 + ( 4 x y ) 2 = 6 2 x 2 + 2 y 2 + 2 x y 8 x 8 y + 10 = 0 x^2+y^2+(4-x-y)^2=6 \Leftrightarrow 2x^2+2y^2+2xy-8x-8y+10=0 .

Rewrite in terms of y y :

2 y 2 + 2 ( x 4 ) y + ( 2 x 2 8 x + 10 ) = 0 2y^2+2(x-4)y+(2x^2-8x+10)=0 (1) .

Δ = ( x 4 ) 2 2 ( 2 x 2 8 x + 10 ) = 3 x 2 + 8 x 4 \Delta'=(x-4)^2-2(2x^2-8x+10)=-3x^2+8x-4 .

Equation (1) has real solutions if and only if 2 3 x 2 \dfrac 23 \leq x \leq 2 .

But when we solve (1) , we have y = 4 x ± 3 x 2 + 8 x 4 2 y=\dfrac{4-x\pm\sqrt{-3x^2+8x-4}}{2} , which is annoying.

So here's what we'll do next. By this method we could generalize the result easily.


Rewrite the system in terms of y , z y,z :

{ y + z = 4 x y 2 + z 2 = 6 x 2 { y + z = 4 x y z = x 2 4 x + 5 \begin{cases} y+z=4-x \\ y^2+z^2=6-x^2 \end{cases} \Leftrightarrow \begin{cases} y+z=4-x \\ yz=x^2-4x+5 \end{cases}

We calculate y + z y+z and y z yz because all symmetric expressions in terms of y y and z z can be expressed as an expression in terms of y + z y+z and y z yz . By this we can avoid roots and calculating mistakes.

For example, y 3 + z 3 = ( y + z ) 3 3 y z ( y + z ) = ( 4 x ) 3 3 ( 4 x ) ( x 2 4 x + 5 ) = 2 x 3 12 x 2 + 15 x + 4 y^3+z^3=(y+z)^3-3yz(y+z)=(4-x)^3-3(4-x)(x^2-4x+5)=2x^3-12x^2+15x+4 .

Hence, P = x 3 + y 3 + z 3 = 3 x 3 12 x 2 + 15 x + 4 P=x^3+y^3+z^3=3x^3-12x^2+15x+4 .

The last part is to find the local maxima and local minima of this expression, using Derivative.

P = 9 x 2 24 x + 15 P'=9x^2-24x+15

P ( 1 ) = P ( 5 3 ) = 0 P'(1)=P'\left(\dfrac53\right)=0

Thus, the result = P ( 1 ) P ( 5 3 ) = 4 9 =\left|P(1)-P\left(\dfrac53\right)\right|=\dfrac49 , and 4 + 9 = 13 4+9=\boxed{13} .

Shaun Leong
Jun 9, 2016

For the sake of algebra:

x + y + z = m x+y+z=m x 2 + y 2 + z 2 = n x^2+y^2+z^2=n x y + y z + x z = m 2 n 2 xy+yz+xz=\frac{m^2-n}{2}

Let b = x + y + z = m b=x+y+z=m , c = x y + y z + x z = m 2 n 2 c=xy+yz+xz=\frac{m^2-n}{2} and d = x y z d=xyz .

x 3 + y 3 + z 3 = 3 x y z + ( x + y + z ) ( x 2 + y 2 + z 2 x y y z x z ) = 3 d + b ( n c ) x^3+y^3+z^3=3xyz+(x+y+z)(x^2+y^2+z^2-xy-yz-xz)=3d+b(n-c)

Note that this expression only has d d as a variable, and b b and c c are constants given values of m m and n n .

Hence P m a x P m i n = 3 ( d m a x d m i n ) P_{max}-P_{min}=3(d_{max}-d_{min})

Consider the cubic polynomial r 3 b r 2 + c r d r^3-br^2+cr-d

x , y x,y and z z are real, so its discriminant is nonnegative.

Δ 1 = b 2 c 2 4 c 3 4 b 3 d 27 d 2 + 18 b c d 0 \Delta_1 = b^2c^2-4c^3-4b^3d-27d^2+18bcd \geq 0

27 d 2 + ( 4 b 3 18 b c ) d + 4 c 3 b 2 c 2 0 27d^2+(4b^3-18bc)d+4c^3-b^2c^2 \leq 0 18 b c 4 b 3 16 b 6 144 b 4 c + 324 b 2 c 2 432 c 3 + 108 b 2 c 2 54 d 18 b c 4 b 3 + 16 b 6 144 b 4 c + 324 b 2 c 2 432 c 3 + 108 b 2 c 2 54 \frac{18bc-4b^3 - \sqrt{16b^6-144b^4c+324b^2c^2-432c^3+108b^2c^2}}{54} \leq d \leq \frac{18bc-4b^3 + \sqrt{16b^6-144b^4c+324b^2c^2-432c^3+108b^2c^2}}{54} d m a x d m i n = 2 16 b 6 144 b 4 c + 432 b 2 c 2 432 c 3 54 d_{max}-d_{min}=\frac{2\sqrt{16b^6-144b^4c+432b^2c^2-432c^3}}{54} P m a x P m i n = 3 ( d m a x d m i n ) = 4 9 b 6 9 b 4 c + 27 b 2 c 2 27 c 3 P_{max}-P_{min}=3(d_{max}-d_{min})=\frac49 \sqrt{b^6-9b^4c+27b^2c^2-27c^3} = 4 9 ( b 2 3 c ) 3 =\frac49 \sqrt{(b^2-3c)^3}

Substituting b = m b=m and c = m 2 n 2 c=\frac{m^2-n}{2} , we get the final answer of 4 9 ( 3 n m 2 2 ) 3 \boxed{\dfrac49 \sqrt{(\dfrac{3n-m^2}{2})^3}}

Hi @Shaun Leong , we really liked your comment, and have converted it into a solution. If you subscribe to this solution, you will receive notifications about future comments.

Brilliant Mathematics Staff - 5 years ago

Log in to reply

I useed AM -GM -HM Thats way easier

Kaustubh Miglani - 4 years, 12 months ago

Log in to reply

Your solution will have an invalid proof, if that's the case. The question only stated that x , y x,y and z z are real numbers, it isn't stated anywhere that they are positive numbers. AM-GM-HM only applies for positive numbers

Hung Woei Neoh - 4 years, 12 months ago

Log in to reply

@Hung Woei Neoh Yes u are right

Kaustubh Miglani - 4 years, 12 months ago

Wonderful generalization

Hung Woei Neoh - 4 years, 12 months ago

Manuel Kahayon
Jun 9, 2016

We use Newton's sums to obtain the ff. equations:

x y + y z + x z = ( x + y + z ) 2 ( x 2 + y 2 + z 2 ) 2 = 4 2 6 2 = 5 xy+yz+xz = \frac{(x+y+z)^2-(x^2+y^2+z^2)}{2} = \frac{4^2-6}{2} = 5 .

x 3 + y 3 + z 3 = ( x + y + z ) ( x 2 + y 2 + z 2 ( x y + y z + z x ) ) + 3 x y z x^{3} + y^{3} + z^{3} = (x+y+z)(x^{2}+y^{2}+z^{2}-(xy+yz+zx)) +3xyz

So, x 3 + y 3 + z 3 x^{3} + y^{3} + z^{3} will be maximized if x y z xyz is maximized and minimized if x y z xyz is minimized.

Letting x , y , z x,y,z be the roots of a cubic equation n 3 ( x + y + z ) n 2 + ( x y + y z + x z ) n x y z = n 3 4 n 2 + 5 n ( x y z ) n^3-(x+y+z)n^2+(xy+yz+xz)n-xyz = n^3-4n^2+5n-(xyz) ,

We see that the graph of f ( n ) f(n) will be either transformed upwards or downwards as x y z xyz increases or decreases respectively. But, in order to have real roots, we must have the x-axis between the graph's local maxima and minima, or exactly on one of them. To maximize x y z xyz , we must have the maxima on the x-axis, and to minimize it, we must have the minima on the x-axis. Either case, the graph is tangent to the x-axis, implying a double root. Having a double root implies WLOG x = z x=z . Therefore, we just substitute this into the equations to obtain 2 x + y = 4 2x+y=4 and 2 x 2 + y 2 = 6 2x^2+y^2 = 6 . Solving gives us either x = z = 1 , y = 2 x=z=1, y=2 or x = z = 5 3 , y = 2 3 x=z=\frac{5}{3}, y = \frac{2}{3} . This gives us values of x 3 + y 3 + z 3 x^3+y^3+z^3 as 10 , 86 9 10, \frac{86}{9} . So, 10 10 is the maxima and 86 9 \frac{86}{9} is the minima. Their difference is 4 9 \frac{4}{9} , so our answer is 4 + 9 = 13 4+9 = \boxed{13}

x y + y z + z x = ( x + y + z ) 2 ( x 2 + y 2 + z 2 2 = 4 2 6 2 = 5 xy + yz + zx = \dfrac{(x+y+z)^{2}-(x^{2}+y^{2}+z^{2}}{2} = \dfrac{4^2 -6}{2} = 5
x 3 + y 3 + z 3 3 x y z = ( x + y + z ) ( x 2 + y 2 + z 2 ( x y + y z + z x ) ) x^{3} + y^{3} + z^{3} - 3xyz = (x+y+z)(x^{2}+y^{2}+z^{2}-(xy+yz+zx))
Let x 3 + y 3 + z 3 = P x^{3} + y^{3} + z^{3} = P
x y z = p 4 3 xyz = \dfrac{p-4}{3}
x , y , z x,y,z are the roots of the equation,
f ( a ) = a 3 ( x + y + z ) a 2 + ( x 2 + b 2 + z 2 ) a ( x y z ) = a 3 4 a 2 + 5 a ( p 4 3 ) f(a) = a^{3} - (x+y+z)a^{2} + (x^{2}+b^{2}+z^{2})a - (xyz) = a^{3} - 4a^{2} + 5a -\left(\dfrac{p-4}{3} \right)
f ( a ) = 3 a 2 8 a + 5 = ( 3 x 5 ) ( x 1 ) f'(a) = 3a^2 - 8a + 5 = (3x-5)(x-1)
The two roots of f ( a ) f'(a) are a = 1 , a = 5 3 a = 1, a = \dfrac{5}{3}
f ( a ) = 6 a 8 f''(a) = 6a-8
f ( 1 ) < 0 a = 1 f''(1) < 0 \to a = 1 is a local maxima.
f ( 5 3 ) > 0 a = 5 3 f''\left(\dfrac{5}{3}\right) > 0 \to a = \dfrac{5}{3} is a local minima.
lim a f ( a ) = , lim a f ( a ) = \displaystyle \lim_{a \to -\infty} f(a)= -\infty , \lim_{a \to \infty}f(a) = \infty
For the equation to have 3 real roots, the maxima must lie above or on the x-axis and the minima must lie below or on the x-axis .
f ( 1 ) 0 \therefore f(1) \geq 0
This gives us,
p 10 p \leq 10
f ( 5 3 ) 0 \therefore f\left(\dfrac{5}{3}\right) \leq 0
This gives us,
p 86 9 p \geq \dfrac{86}{9}
86 9 x 3 + y 3 + z 3 10 \dfrac{86}{9} \leq x^{3} + y^{3} + z^{3} \leq 10
p m a x p m i n = 10 86 9 = 4 9 p_{max} - p_{min} = 10 - \dfrac{86}{9} = \dfrac{4}{9}
a + b = 4 + 9 = 13 a + b = 4 + 9 = 13






P m a x P_{max} occurs when x , y , z x,y,z are a permutation of roots of,
f ( a ) = a 3 4 a 2 + 5 a 2 = ( a 1 ) 3 ( a 2 ) = 0 ( x , y , z ) ( 1 , 1 , 2 ) f(a) = a^3 - 4a^2 + 5a - 2 = (a-1)^{3}(a-2) = 0 \to (x,y,z) \in (1,1,2)
P m i n P_{min} occurs when x , y , z x,y,z are a permutation of roots of,
f ( a ) = a 3 4 a 2 + 5 a 50 27 = ( a 5 3 ) 2 ( a 2 3 ) = 0 ( x , y , z ) ( 5 3 , 5 3 , 2 3 ) f(a) = a^3 - 4a^2 + 5a - \dfrac{50}{27} = (a-\dfrac{5}{3})^{2}(a-\dfrac{2}{3}) = 0 \to (x,y,z) \in ( \dfrac{5}{3}, \dfrac{5}{3}, \dfrac{2}{3})

0 pending reports

×

Problem Loading...

Note Loading...

Set Loading...